What comes after $cosleft(tfrac2pi7right)^1/3+cosleft(tfrac4pi7right)^1/3+cosleft(tfrac6pi7right)^1/3$?

The name of the pictureThe name of the pictureThe name of the pictureClash Royale CLAN TAG#URR8PPP











up vote
22
down vote

favorite
13












We have,



$$big(cos(tfrac2pi5)^1/2+(-cos(tfrac4pi5))^1/2big)^2 = tfrac12left(tfrac-1+sqrt52right)^3tag1$$



$$big(cos(tfrac2pi7)^1/3+cos(tfrac4pi7)^1/3+cos(tfrac6pi7)^1/3big)^3 = frac5-3cdot 7^1/32tag2$$



$$big(cos(tfrac2pi11)^1/5+cos(tfrac4pi11)^1/5+dots+cos(tfrac10pi11)^1/5big)^5 = x?tag3$$



Question: What degree is the minimal polynomial of $x$? Since the previous two are deg 2 and 3, I had assumed (3) would be deg 5, but Mathematica does not recognize it as a quintic with small coefficients, nor a 25th deg (even after using 500-decimal digit precision, though I am not sure of the latter result).







share|cite|improve this question






















  • How have you derived the identities? For cubic, we can set $y=(frac z2)^frac13$ here in my answer math.stackexchange.com/questions/225088/… and then use Vieta's formula
    – lab bhattacharjee
    Dec 19 '13 at 5:53











  • The first is easy, while the second is at the end of mathworld.wolfram.com/TrigonometryAnglesPi7.html. The third is unknown.
    – Tito Piezas III
    Dec 19 '13 at 16:19










  • Are you sure about the first "identity"? In particular, $cos frac4pi5$ is negative, so taking square root of it results in purely imaginary number; so the sum of two terms is neither real, nor purely imaginary... and thus its square cannot be real.
    – Peter KoÅ¡inár
    Dec 19 '13 at 22:40






  • 1




    @GrigoryM: What Ramanujan formula? Regarding (3), since $y^5=a_i$ has five roots, and we are to do so for five $a_i=cos(n)$, then I think it has maximum degree $5^5 = 3125$ though it may factor since the $a_i$ are not random. I tested the form $z=(y_1+y_2zeta^k+y_3zeta^2k+y_4zeta^3k+y_5zeta^4k)^5$ where the $y_i = cos(n)^1/5$ and $zeta^5=1$ and tried to form a 125-deg resolvent, but it still doesn't seem to be the minimal deg.
    – Tito Piezas III
    Dec 20 '13 at 20:03






  • 1




    I'm not sure there exists a (good) generalization in this direction — but maybe you'll find another generalization of the second formula of interest
    – Grigory M
    Jan 20 '14 at 8:33














up vote
22
down vote

favorite
13












We have,



$$big(cos(tfrac2pi5)^1/2+(-cos(tfrac4pi5))^1/2big)^2 = tfrac12left(tfrac-1+sqrt52right)^3tag1$$



$$big(cos(tfrac2pi7)^1/3+cos(tfrac4pi7)^1/3+cos(tfrac6pi7)^1/3big)^3 = frac5-3cdot 7^1/32tag2$$



$$big(cos(tfrac2pi11)^1/5+cos(tfrac4pi11)^1/5+dots+cos(tfrac10pi11)^1/5big)^5 = x?tag3$$



Question: What degree is the minimal polynomial of $x$? Since the previous two are deg 2 and 3, I had assumed (3) would be deg 5, but Mathematica does not recognize it as a quintic with small coefficients, nor a 25th deg (even after using 500-decimal digit precision, though I am not sure of the latter result).







share|cite|improve this question






















  • How have you derived the identities? For cubic, we can set $y=(frac z2)^frac13$ here in my answer math.stackexchange.com/questions/225088/… and then use Vieta's formula
    – lab bhattacharjee
    Dec 19 '13 at 5:53











  • The first is easy, while the second is at the end of mathworld.wolfram.com/TrigonometryAnglesPi7.html. The third is unknown.
    – Tito Piezas III
    Dec 19 '13 at 16:19










  • Are you sure about the first "identity"? In particular, $cos frac4pi5$ is negative, so taking square root of it results in purely imaginary number; so the sum of two terms is neither real, nor purely imaginary... and thus its square cannot be real.
    – Peter KoÅ¡inár
    Dec 19 '13 at 22:40






  • 1




    @GrigoryM: What Ramanujan formula? Regarding (3), since $y^5=a_i$ has five roots, and we are to do so for five $a_i=cos(n)$, then I think it has maximum degree $5^5 = 3125$ though it may factor since the $a_i$ are not random. I tested the form $z=(y_1+y_2zeta^k+y_3zeta^2k+y_4zeta^3k+y_5zeta^4k)^5$ where the $y_i = cos(n)^1/5$ and $zeta^5=1$ and tried to form a 125-deg resolvent, but it still doesn't seem to be the minimal deg.
    – Tito Piezas III
    Dec 20 '13 at 20:03






  • 1




    I'm not sure there exists a (good) generalization in this direction — but maybe you'll find another generalization of the second formula of interest
    – Grigory M
    Jan 20 '14 at 8:33












up vote
22
down vote

favorite
13









up vote
22
down vote

favorite
13






13





We have,



$$big(cos(tfrac2pi5)^1/2+(-cos(tfrac4pi5))^1/2big)^2 = tfrac12left(tfrac-1+sqrt52right)^3tag1$$



$$big(cos(tfrac2pi7)^1/3+cos(tfrac4pi7)^1/3+cos(tfrac6pi7)^1/3big)^3 = frac5-3cdot 7^1/32tag2$$



$$big(cos(tfrac2pi11)^1/5+cos(tfrac4pi11)^1/5+dots+cos(tfrac10pi11)^1/5big)^5 = x?tag3$$



Question: What degree is the minimal polynomial of $x$? Since the previous two are deg 2 and 3, I had assumed (3) would be deg 5, but Mathematica does not recognize it as a quintic with small coefficients, nor a 25th deg (even after using 500-decimal digit precision, though I am not sure of the latter result).







share|cite|improve this question














We have,



$$big(cos(tfrac2pi5)^1/2+(-cos(tfrac4pi5))^1/2big)^2 = tfrac12left(tfrac-1+sqrt52right)^3tag1$$



$$big(cos(tfrac2pi7)^1/3+cos(tfrac4pi7)^1/3+cos(tfrac6pi7)^1/3big)^3 = frac5-3cdot 7^1/32tag2$$



$$big(cos(tfrac2pi11)^1/5+cos(tfrac4pi11)^1/5+dots+cos(tfrac10pi11)^1/5big)^5 = x?tag3$$



Question: What degree is the minimal polynomial of $x$? Since the previous two are deg 2 and 3, I had assumed (3) would be deg 5, but Mathematica does not recognize it as a quintic with small coefficients, nor a 25th deg (even after using 500-decimal digit precision, though I am not sure of the latter result).









share|cite|improve this question













share|cite|improve this question




share|cite|improve this question








edited Apr 30 at 8:37









mathreadler

13.7k71957




13.7k71957










asked Dec 19 '13 at 5:26









Tito Piezas III

26.2k361161




26.2k361161











  • How have you derived the identities? For cubic, we can set $y=(frac z2)^frac13$ here in my answer math.stackexchange.com/questions/225088/… and then use Vieta's formula
    – lab bhattacharjee
    Dec 19 '13 at 5:53











  • The first is easy, while the second is at the end of mathworld.wolfram.com/TrigonometryAnglesPi7.html. The third is unknown.
    – Tito Piezas III
    Dec 19 '13 at 16:19










  • Are you sure about the first "identity"? In particular, $cos frac4pi5$ is negative, so taking square root of it results in purely imaginary number; so the sum of two terms is neither real, nor purely imaginary... and thus its square cannot be real.
    – Peter KoÅ¡inár
    Dec 19 '13 at 22:40






  • 1




    @GrigoryM: What Ramanujan formula? Regarding (3), since $y^5=a_i$ has five roots, and we are to do so for five $a_i=cos(n)$, then I think it has maximum degree $5^5 = 3125$ though it may factor since the $a_i$ are not random. I tested the form $z=(y_1+y_2zeta^k+y_3zeta^2k+y_4zeta^3k+y_5zeta^4k)^5$ where the $y_i = cos(n)^1/5$ and $zeta^5=1$ and tried to form a 125-deg resolvent, but it still doesn't seem to be the minimal deg.
    – Tito Piezas III
    Dec 20 '13 at 20:03






  • 1




    I'm not sure there exists a (good) generalization in this direction — but maybe you'll find another generalization of the second formula of interest
    – Grigory M
    Jan 20 '14 at 8:33
















  • How have you derived the identities? For cubic, we can set $y=(frac z2)^frac13$ here in my answer math.stackexchange.com/questions/225088/… and then use Vieta's formula
    – lab bhattacharjee
    Dec 19 '13 at 5:53











  • The first is easy, while the second is at the end of mathworld.wolfram.com/TrigonometryAnglesPi7.html. The third is unknown.
    – Tito Piezas III
    Dec 19 '13 at 16:19










  • Are you sure about the first "identity"? In particular, $cos frac4pi5$ is negative, so taking square root of it results in purely imaginary number; so the sum of two terms is neither real, nor purely imaginary... and thus its square cannot be real.
    – Peter KoÅ¡inár
    Dec 19 '13 at 22:40






  • 1




    @GrigoryM: What Ramanujan formula? Regarding (3), since $y^5=a_i$ has five roots, and we are to do so for five $a_i=cos(n)$, then I think it has maximum degree $5^5 = 3125$ though it may factor since the $a_i$ are not random. I tested the form $z=(y_1+y_2zeta^k+y_3zeta^2k+y_4zeta^3k+y_5zeta^4k)^5$ where the $y_i = cos(n)^1/5$ and $zeta^5=1$ and tried to form a 125-deg resolvent, but it still doesn't seem to be the minimal deg.
    – Tito Piezas III
    Dec 20 '13 at 20:03






  • 1




    I'm not sure there exists a (good) generalization in this direction — but maybe you'll find another generalization of the second formula of interest
    – Grigory M
    Jan 20 '14 at 8:33















How have you derived the identities? For cubic, we can set $y=(frac z2)^frac13$ here in my answer math.stackexchange.com/questions/225088/… and then use Vieta's formula
– lab bhattacharjee
Dec 19 '13 at 5:53





How have you derived the identities? For cubic, we can set $y=(frac z2)^frac13$ here in my answer math.stackexchange.com/questions/225088/… and then use Vieta's formula
– lab bhattacharjee
Dec 19 '13 at 5:53













The first is easy, while the second is at the end of mathworld.wolfram.com/TrigonometryAnglesPi7.html. The third is unknown.
– Tito Piezas III
Dec 19 '13 at 16:19




The first is easy, while the second is at the end of mathworld.wolfram.com/TrigonometryAnglesPi7.html. The third is unknown.
– Tito Piezas III
Dec 19 '13 at 16:19












Are you sure about the first "identity"? In particular, $cos frac4pi5$ is negative, so taking square root of it results in purely imaginary number; so the sum of two terms is neither real, nor purely imaginary... and thus its square cannot be real.
– Peter KoÅ¡inár
Dec 19 '13 at 22:40




Are you sure about the first "identity"? In particular, $cos frac4pi5$ is negative, so taking square root of it results in purely imaginary number; so the sum of two terms is neither real, nor purely imaginary... and thus its square cannot be real.
– Peter KoÅ¡inár
Dec 19 '13 at 22:40




1




1




@GrigoryM: What Ramanujan formula? Regarding (3), since $y^5=a_i$ has five roots, and we are to do so for five $a_i=cos(n)$, then I think it has maximum degree $5^5 = 3125$ though it may factor since the $a_i$ are not random. I tested the form $z=(y_1+y_2zeta^k+y_3zeta^2k+y_4zeta^3k+y_5zeta^4k)^5$ where the $y_i = cos(n)^1/5$ and $zeta^5=1$ and tried to form a 125-deg resolvent, but it still doesn't seem to be the minimal deg.
– Tito Piezas III
Dec 20 '13 at 20:03




@GrigoryM: What Ramanujan formula? Regarding (3), since $y^5=a_i$ has five roots, and we are to do so for five $a_i=cos(n)$, then I think it has maximum degree $5^5 = 3125$ though it may factor since the $a_i$ are not random. I tested the form $z=(y_1+y_2zeta^k+y_3zeta^2k+y_4zeta^3k+y_5zeta^4k)^5$ where the $y_i = cos(n)^1/5$ and $zeta^5=1$ and tried to form a 125-deg resolvent, but it still doesn't seem to be the minimal deg.
– Tito Piezas III
Dec 20 '13 at 20:03




1




1




I'm not sure there exists a (good) generalization in this direction — but maybe you'll find another generalization of the second formula of interest
– Grigory M
Jan 20 '14 at 8:33




I'm not sure there exists a (good) generalization in this direction — but maybe you'll find another generalization of the second formula of interest
– Grigory M
Jan 20 '14 at 8:33










1 Answer
1






active

oldest

votes

















up vote
2
down vote













As some people said, the 2nd formula is easy to derive. In Maple, there are commands to get the minimal polynomial of LHS of the 2nd. The following method works for the 2nd formula, but not for the 3rd.



Prove that $sqrt[3]cosfrac2pi7 + sqrt[3]cosfrac4pi7 + sqrt[3]cosfrac6pi7 = sqrt[3]frac5-3sqrt[3]72$.



$textitProof$: $cosfrac2pi7,cosfrac4pi7,cosfrac6pi7$ are the three distinct real roots of $8x^3+4x^2-4x-1=0$.
Let $u_1=sqrt[3]cosfrac2pi7, u_2=sqrt[3]cosfrac4pi7, u_3=sqrt[3]cosfrac6pi7$ which are all real.
All roots of $8u^9+4u^6-4u^3-1=0$ are $u_1, t u_1, t^2 u_1, u_2, t u_2, t^2 u_2, u_3, t u_3, t^2 u_3$
where $t = e^2pi i/3$.



Let $(u-u_1)(u-u_2)(u-u_3)=u^3+a_2u^2+a_1u - 1/2$. We have
beginalign
(u-tu_1)(u-tu_2)(u-tu_3) &=u^3+ta_2u^2+t^2a_1u - 1/2, \
(u-t^2u_1)(u-t^2u_2)(u-t^2u_3) &=u^3+t^2a_2u^2+ta_1u - 1/2.
endalign
It follows from
beginalign
&8u^9+4u^6-4u^3-1 \
= & 8(u^3+a_2u^2+a_1u - 1/2)(u^3+ta_2u^2+t^2a_1u - 1/2)(u^3+t^2a_2u^2+ta_1u - 1/2)
endalign that
beginalign
a_2^3-3a_1a_2-2 &=0, \
4a_1^3+6a_1a_2+5 &= 0.
endalign
From these two equations, we get $a_1 = fraca_2^3-23a_2$
and $4a_2^9+30a_2^6+75a_2^3-32=0$.
Let $f(v)=4v^9+30v^6+75v^3-32$. We have $f'(v)=9v^2(2v^3+5)^2ge 0$. Thus, $f(v)=0$ has a unique real root.
It is easy to check that $v=-sqrt[3]frac5-3sqrt[3]72$ is a root of
$f(v)=0$. Thus, $a_2 = -sqrt[3]frac5-3sqrt[3]72$.
Thus, $u_1+u_2+u_3 = -a_2 = sqrt[3]frac5-3sqrt[3]72$. This completes the proof.



In general, suppose $x_1,x_2,x_3$ are the three distinct real roots of $x^3+bx^2+cx+d^3=0$,
where $b,c,d$ are rational numbers. We have
beginequation
sqrt[3]x_1+sqrt[3]x_2+sqrt[3]x_3 = sqrt[3]
-frac32Big(sqrt[3]Q+4sqrtDelta+sqrt[3]Q-4sqrtDeltaBig) -b-6d ,
endequation
where $Q = 24bd^2+36d^3+4bc+24cd, Delta = -4b^3d^3-27d^6+18bcd^3+b^2c^2-4c^3$.
Here, $Delta$ is the discriminant of $x^3+bx^2+cx+d^3=0$.



For the 3rd, similarly,
$x_k = cosfrac2kpi11, k=1,cdots, 5$ are the five distinct real roots of $32x^5+16x^4-32x^3-12x^2+6x+1=0$.
Let $u_1 = sqrt[5]cosfrac2pi11, u_2= sqrt[5]cosfrac4pi11, u_3 = sqrt[5]cosfrac6pi11, u_4 = sqrt[5]cosfrac8pi11, u_5= sqrt[5]cosfrac10pi11$.
All roots of $32u^25+16u^20-32u^15-12u^10+6u^5+1=0$ are
$t^j u_k, k=1,2,3,4, 5; j=0,1,2,3,4$ where $t= e^2pi i/5$.



Let $prod_k=1^5 (u-u_k) = u^5 + a_4u^4+a_3u^3+a_2u^2+a_1u+frac12$.
Similarly, we have four equations of
beginalign
0 & = a_4^5-5a_3a_4^3+5a_2a_4^2+5a_3^2a_4-5a_1a_4-5a_2a_3+2, \
0 & = 8a_1^5-20a_1^3a_2+10a_1^2a_3+10a_1a_2^2-5a_1a_4-5a_2a_3+1,\
0 & = -10a_1a_2a_4^3+10a_1a_3^2a_4^2+10a_2^2a_3a_4^2-10a_2a_3^3a_4+2a_3^5+10a_1^2a_4^2-10a_1a_2a_3a_4\
&quad-10a_1a_3^3-10a_2^3a_4+10a_2^2a_3^2-5a_3a_4^3+10a_1^2a_3+10a_1a_2^2+10a_2a_4^2+10a_3^2a_4\
&quad -15a_1a_4-15a_2a_3+7,\
0 & = -40a_1^3a_3a_4+40a_1^2a_2^2a_4+40a_1^2a_2a_3^2-40a_1a_2^3a_3+8a_2^5-40a_1^3a_2+20a_1^2a_4^2\
&quad-20a_1a_2a_3a_4-20a_1a_3^3-20a_2^3a_4+20a_2^2a_3^2+40a_1^2a_3+40a_1a_2^2+10a_2a_4^2\
&quad +10a_3^2a_4-30a_1a_4-30a_2a_3+13.
endalign



However, it seems no hope for determination of $a_4$.






share|cite|improve this answer


















  • 2




    I don't mean to be rude, by I find this utter unreadable. Could you perhaps make use of some displayed math environments (using $$ (math here) $$) or the align environment (beginalign endalign) to make it easier to follow? You can also find other formatting tips in the MathJax tutorial.
    – Xander Henderson
    Aug 17 at 3:19










Your Answer




StackExchange.ifUsing("editor", function ()
return StackExchange.using("mathjaxEditing", function ()
StackExchange.MarkdownEditor.creationCallbacks.add(function (editor, postfix)
StackExchange.mathjaxEditing.prepareWmdForMathJax(editor, postfix, [["$", "$"], ["\\(","\\)"]]);
);
);
, "mathjax-editing");

StackExchange.ready(function()
var channelOptions =
tags: "".split(" "),
id: "69"
;
initTagRenderer("".split(" "), "".split(" "), channelOptions);

StackExchange.using("externalEditor", function()
// Have to fire editor after snippets, if snippets enabled
if (StackExchange.settings.snippets.snippetsEnabled)
StackExchange.using("snippets", function()
createEditor();
);

else
createEditor();

);

function createEditor()
StackExchange.prepareEditor(
heartbeatType: 'answer',
convertImagesToLinks: true,
noModals: false,
showLowRepImageUploadWarning: true,
reputationToPostImages: 10,
bindNavPrevention: true,
postfix: "",
noCode: true, onDemand: true,
discardSelector: ".discard-answer"
,immediatelyShowMarkdownHelp:true
);



);








 

draft saved


draft discarded


















StackExchange.ready(
function ()
StackExchange.openid.initPostLogin('.new-post-login', 'https%3a%2f%2fmath.stackexchange.com%2fquestions%2f612607%2fwhat-comes-after-cos-left-tfrac2-pi7-right1-3-cos-left-tfrac4-pi%23new-answer', 'question_page');

);

Post as a guest






























1 Answer
1






active

oldest

votes








1 Answer
1






active

oldest

votes









active

oldest

votes






active

oldest

votes








up vote
2
down vote













As some people said, the 2nd formula is easy to derive. In Maple, there are commands to get the minimal polynomial of LHS of the 2nd. The following method works for the 2nd formula, but not for the 3rd.



Prove that $sqrt[3]cosfrac2pi7 + sqrt[3]cosfrac4pi7 + sqrt[3]cosfrac6pi7 = sqrt[3]frac5-3sqrt[3]72$.



$textitProof$: $cosfrac2pi7,cosfrac4pi7,cosfrac6pi7$ are the three distinct real roots of $8x^3+4x^2-4x-1=0$.
Let $u_1=sqrt[3]cosfrac2pi7, u_2=sqrt[3]cosfrac4pi7, u_3=sqrt[3]cosfrac6pi7$ which are all real.
All roots of $8u^9+4u^6-4u^3-1=0$ are $u_1, t u_1, t^2 u_1, u_2, t u_2, t^2 u_2, u_3, t u_3, t^2 u_3$
where $t = e^2pi i/3$.



Let $(u-u_1)(u-u_2)(u-u_3)=u^3+a_2u^2+a_1u - 1/2$. We have
beginalign
(u-tu_1)(u-tu_2)(u-tu_3) &=u^3+ta_2u^2+t^2a_1u - 1/2, \
(u-t^2u_1)(u-t^2u_2)(u-t^2u_3) &=u^3+t^2a_2u^2+ta_1u - 1/2.
endalign
It follows from
beginalign
&8u^9+4u^6-4u^3-1 \
= & 8(u^3+a_2u^2+a_1u - 1/2)(u^3+ta_2u^2+t^2a_1u - 1/2)(u^3+t^2a_2u^2+ta_1u - 1/2)
endalign that
beginalign
a_2^3-3a_1a_2-2 &=0, \
4a_1^3+6a_1a_2+5 &= 0.
endalign
From these two equations, we get $a_1 = fraca_2^3-23a_2$
and $4a_2^9+30a_2^6+75a_2^3-32=0$.
Let $f(v)=4v^9+30v^6+75v^3-32$. We have $f'(v)=9v^2(2v^3+5)^2ge 0$. Thus, $f(v)=0$ has a unique real root.
It is easy to check that $v=-sqrt[3]frac5-3sqrt[3]72$ is a root of
$f(v)=0$. Thus, $a_2 = -sqrt[3]frac5-3sqrt[3]72$.
Thus, $u_1+u_2+u_3 = -a_2 = sqrt[3]frac5-3sqrt[3]72$. This completes the proof.



In general, suppose $x_1,x_2,x_3$ are the three distinct real roots of $x^3+bx^2+cx+d^3=0$,
where $b,c,d$ are rational numbers. We have
beginequation
sqrt[3]x_1+sqrt[3]x_2+sqrt[3]x_3 = sqrt[3]
-frac32Big(sqrt[3]Q+4sqrtDelta+sqrt[3]Q-4sqrtDeltaBig) -b-6d ,
endequation
where $Q = 24bd^2+36d^3+4bc+24cd, Delta = -4b^3d^3-27d^6+18bcd^3+b^2c^2-4c^3$.
Here, $Delta$ is the discriminant of $x^3+bx^2+cx+d^3=0$.



For the 3rd, similarly,
$x_k = cosfrac2kpi11, k=1,cdots, 5$ are the five distinct real roots of $32x^5+16x^4-32x^3-12x^2+6x+1=0$.
Let $u_1 = sqrt[5]cosfrac2pi11, u_2= sqrt[5]cosfrac4pi11, u_3 = sqrt[5]cosfrac6pi11, u_4 = sqrt[5]cosfrac8pi11, u_5= sqrt[5]cosfrac10pi11$.
All roots of $32u^25+16u^20-32u^15-12u^10+6u^5+1=0$ are
$t^j u_k, k=1,2,3,4, 5; j=0,1,2,3,4$ where $t= e^2pi i/5$.



Let $prod_k=1^5 (u-u_k) = u^5 + a_4u^4+a_3u^3+a_2u^2+a_1u+frac12$.
Similarly, we have four equations of
beginalign
0 & = a_4^5-5a_3a_4^3+5a_2a_4^2+5a_3^2a_4-5a_1a_4-5a_2a_3+2, \
0 & = 8a_1^5-20a_1^3a_2+10a_1^2a_3+10a_1a_2^2-5a_1a_4-5a_2a_3+1,\
0 & = -10a_1a_2a_4^3+10a_1a_3^2a_4^2+10a_2^2a_3a_4^2-10a_2a_3^3a_4+2a_3^5+10a_1^2a_4^2-10a_1a_2a_3a_4\
&quad-10a_1a_3^3-10a_2^3a_4+10a_2^2a_3^2-5a_3a_4^3+10a_1^2a_3+10a_1a_2^2+10a_2a_4^2+10a_3^2a_4\
&quad -15a_1a_4-15a_2a_3+7,\
0 & = -40a_1^3a_3a_4+40a_1^2a_2^2a_4+40a_1^2a_2a_3^2-40a_1a_2^3a_3+8a_2^5-40a_1^3a_2+20a_1^2a_4^2\
&quad-20a_1a_2a_3a_4-20a_1a_3^3-20a_2^3a_4+20a_2^2a_3^2+40a_1^2a_3+40a_1a_2^2+10a_2a_4^2\
&quad +10a_3^2a_4-30a_1a_4-30a_2a_3+13.
endalign



However, it seems no hope for determination of $a_4$.






share|cite|improve this answer


















  • 2




    I don't mean to be rude, by I find this utter unreadable. Could you perhaps make use of some displayed math environments (using $$ (math here) $$) or the align environment (beginalign endalign) to make it easier to follow? You can also find other formatting tips in the MathJax tutorial.
    – Xander Henderson
    Aug 17 at 3:19














up vote
2
down vote













As some people said, the 2nd formula is easy to derive. In Maple, there are commands to get the minimal polynomial of LHS of the 2nd. The following method works for the 2nd formula, but not for the 3rd.



Prove that $sqrt[3]cosfrac2pi7 + sqrt[3]cosfrac4pi7 + sqrt[3]cosfrac6pi7 = sqrt[3]frac5-3sqrt[3]72$.



$textitProof$: $cosfrac2pi7,cosfrac4pi7,cosfrac6pi7$ are the three distinct real roots of $8x^3+4x^2-4x-1=0$.
Let $u_1=sqrt[3]cosfrac2pi7, u_2=sqrt[3]cosfrac4pi7, u_3=sqrt[3]cosfrac6pi7$ which are all real.
All roots of $8u^9+4u^6-4u^3-1=0$ are $u_1, t u_1, t^2 u_1, u_2, t u_2, t^2 u_2, u_3, t u_3, t^2 u_3$
where $t = e^2pi i/3$.



Let $(u-u_1)(u-u_2)(u-u_3)=u^3+a_2u^2+a_1u - 1/2$. We have
beginalign
(u-tu_1)(u-tu_2)(u-tu_3) &=u^3+ta_2u^2+t^2a_1u - 1/2, \
(u-t^2u_1)(u-t^2u_2)(u-t^2u_3) &=u^3+t^2a_2u^2+ta_1u - 1/2.
endalign
It follows from
beginalign
&8u^9+4u^6-4u^3-1 \
= & 8(u^3+a_2u^2+a_1u - 1/2)(u^3+ta_2u^2+t^2a_1u - 1/2)(u^3+t^2a_2u^2+ta_1u - 1/2)
endalign that
beginalign
a_2^3-3a_1a_2-2 &=0, \
4a_1^3+6a_1a_2+5 &= 0.
endalign
From these two equations, we get $a_1 = fraca_2^3-23a_2$
and $4a_2^9+30a_2^6+75a_2^3-32=0$.
Let $f(v)=4v^9+30v^6+75v^3-32$. We have $f'(v)=9v^2(2v^3+5)^2ge 0$. Thus, $f(v)=0$ has a unique real root.
It is easy to check that $v=-sqrt[3]frac5-3sqrt[3]72$ is a root of
$f(v)=0$. Thus, $a_2 = -sqrt[3]frac5-3sqrt[3]72$.
Thus, $u_1+u_2+u_3 = -a_2 = sqrt[3]frac5-3sqrt[3]72$. This completes the proof.



In general, suppose $x_1,x_2,x_3$ are the three distinct real roots of $x^3+bx^2+cx+d^3=0$,
where $b,c,d$ are rational numbers. We have
beginequation
sqrt[3]x_1+sqrt[3]x_2+sqrt[3]x_3 = sqrt[3]
-frac32Big(sqrt[3]Q+4sqrtDelta+sqrt[3]Q-4sqrtDeltaBig) -b-6d ,
endequation
where $Q = 24bd^2+36d^3+4bc+24cd, Delta = -4b^3d^3-27d^6+18bcd^3+b^2c^2-4c^3$.
Here, $Delta$ is the discriminant of $x^3+bx^2+cx+d^3=0$.



For the 3rd, similarly,
$x_k = cosfrac2kpi11, k=1,cdots, 5$ are the five distinct real roots of $32x^5+16x^4-32x^3-12x^2+6x+1=0$.
Let $u_1 = sqrt[5]cosfrac2pi11, u_2= sqrt[5]cosfrac4pi11, u_3 = sqrt[5]cosfrac6pi11, u_4 = sqrt[5]cosfrac8pi11, u_5= sqrt[5]cosfrac10pi11$.
All roots of $32u^25+16u^20-32u^15-12u^10+6u^5+1=0$ are
$t^j u_k, k=1,2,3,4, 5; j=0,1,2,3,4$ where $t= e^2pi i/5$.



Let $prod_k=1^5 (u-u_k) = u^5 + a_4u^4+a_3u^3+a_2u^2+a_1u+frac12$.
Similarly, we have four equations of
beginalign
0 & = a_4^5-5a_3a_4^3+5a_2a_4^2+5a_3^2a_4-5a_1a_4-5a_2a_3+2, \
0 & = 8a_1^5-20a_1^3a_2+10a_1^2a_3+10a_1a_2^2-5a_1a_4-5a_2a_3+1,\
0 & = -10a_1a_2a_4^3+10a_1a_3^2a_4^2+10a_2^2a_3a_4^2-10a_2a_3^3a_4+2a_3^5+10a_1^2a_4^2-10a_1a_2a_3a_4\
&quad-10a_1a_3^3-10a_2^3a_4+10a_2^2a_3^2-5a_3a_4^3+10a_1^2a_3+10a_1a_2^2+10a_2a_4^2+10a_3^2a_4\
&quad -15a_1a_4-15a_2a_3+7,\
0 & = -40a_1^3a_3a_4+40a_1^2a_2^2a_4+40a_1^2a_2a_3^2-40a_1a_2^3a_3+8a_2^5-40a_1^3a_2+20a_1^2a_4^2\
&quad-20a_1a_2a_3a_4-20a_1a_3^3-20a_2^3a_4+20a_2^2a_3^2+40a_1^2a_3+40a_1a_2^2+10a_2a_4^2\
&quad +10a_3^2a_4-30a_1a_4-30a_2a_3+13.
endalign



However, it seems no hope for determination of $a_4$.






share|cite|improve this answer


















  • 2




    I don't mean to be rude, by I find this utter unreadable. Could you perhaps make use of some displayed math environments (using $$ (math here) $$) or the align environment (beginalign endalign) to make it easier to follow? You can also find other formatting tips in the MathJax tutorial.
    – Xander Henderson
    Aug 17 at 3:19












up vote
2
down vote










up vote
2
down vote









As some people said, the 2nd formula is easy to derive. In Maple, there are commands to get the minimal polynomial of LHS of the 2nd. The following method works for the 2nd formula, but not for the 3rd.



Prove that $sqrt[3]cosfrac2pi7 + sqrt[3]cosfrac4pi7 + sqrt[3]cosfrac6pi7 = sqrt[3]frac5-3sqrt[3]72$.



$textitProof$: $cosfrac2pi7,cosfrac4pi7,cosfrac6pi7$ are the three distinct real roots of $8x^3+4x^2-4x-1=0$.
Let $u_1=sqrt[3]cosfrac2pi7, u_2=sqrt[3]cosfrac4pi7, u_3=sqrt[3]cosfrac6pi7$ which are all real.
All roots of $8u^9+4u^6-4u^3-1=0$ are $u_1, t u_1, t^2 u_1, u_2, t u_2, t^2 u_2, u_3, t u_3, t^2 u_3$
where $t = e^2pi i/3$.



Let $(u-u_1)(u-u_2)(u-u_3)=u^3+a_2u^2+a_1u - 1/2$. We have
beginalign
(u-tu_1)(u-tu_2)(u-tu_3) &=u^3+ta_2u^2+t^2a_1u - 1/2, \
(u-t^2u_1)(u-t^2u_2)(u-t^2u_3) &=u^3+t^2a_2u^2+ta_1u - 1/2.
endalign
It follows from
beginalign
&8u^9+4u^6-4u^3-1 \
= & 8(u^3+a_2u^2+a_1u - 1/2)(u^3+ta_2u^2+t^2a_1u - 1/2)(u^3+t^2a_2u^2+ta_1u - 1/2)
endalign that
beginalign
a_2^3-3a_1a_2-2 &=0, \
4a_1^3+6a_1a_2+5 &= 0.
endalign
From these two equations, we get $a_1 = fraca_2^3-23a_2$
and $4a_2^9+30a_2^6+75a_2^3-32=0$.
Let $f(v)=4v^9+30v^6+75v^3-32$. We have $f'(v)=9v^2(2v^3+5)^2ge 0$. Thus, $f(v)=0$ has a unique real root.
It is easy to check that $v=-sqrt[3]frac5-3sqrt[3]72$ is a root of
$f(v)=0$. Thus, $a_2 = -sqrt[3]frac5-3sqrt[3]72$.
Thus, $u_1+u_2+u_3 = -a_2 = sqrt[3]frac5-3sqrt[3]72$. This completes the proof.



In general, suppose $x_1,x_2,x_3$ are the three distinct real roots of $x^3+bx^2+cx+d^3=0$,
where $b,c,d$ are rational numbers. We have
beginequation
sqrt[3]x_1+sqrt[3]x_2+sqrt[3]x_3 = sqrt[3]
-frac32Big(sqrt[3]Q+4sqrtDelta+sqrt[3]Q-4sqrtDeltaBig) -b-6d ,
endequation
where $Q = 24bd^2+36d^3+4bc+24cd, Delta = -4b^3d^3-27d^6+18bcd^3+b^2c^2-4c^3$.
Here, $Delta$ is the discriminant of $x^3+bx^2+cx+d^3=0$.



For the 3rd, similarly,
$x_k = cosfrac2kpi11, k=1,cdots, 5$ are the five distinct real roots of $32x^5+16x^4-32x^3-12x^2+6x+1=0$.
Let $u_1 = sqrt[5]cosfrac2pi11, u_2= sqrt[5]cosfrac4pi11, u_3 = sqrt[5]cosfrac6pi11, u_4 = sqrt[5]cosfrac8pi11, u_5= sqrt[5]cosfrac10pi11$.
All roots of $32u^25+16u^20-32u^15-12u^10+6u^5+1=0$ are
$t^j u_k, k=1,2,3,4, 5; j=0,1,2,3,4$ where $t= e^2pi i/5$.



Let $prod_k=1^5 (u-u_k) = u^5 + a_4u^4+a_3u^3+a_2u^2+a_1u+frac12$.
Similarly, we have four equations of
beginalign
0 & = a_4^5-5a_3a_4^3+5a_2a_4^2+5a_3^2a_4-5a_1a_4-5a_2a_3+2, \
0 & = 8a_1^5-20a_1^3a_2+10a_1^2a_3+10a_1a_2^2-5a_1a_4-5a_2a_3+1,\
0 & = -10a_1a_2a_4^3+10a_1a_3^2a_4^2+10a_2^2a_3a_4^2-10a_2a_3^3a_4+2a_3^5+10a_1^2a_4^2-10a_1a_2a_3a_4\
&quad-10a_1a_3^3-10a_2^3a_4+10a_2^2a_3^2-5a_3a_4^3+10a_1^2a_3+10a_1a_2^2+10a_2a_4^2+10a_3^2a_4\
&quad -15a_1a_4-15a_2a_3+7,\
0 & = -40a_1^3a_3a_4+40a_1^2a_2^2a_4+40a_1^2a_2a_3^2-40a_1a_2^3a_3+8a_2^5-40a_1^3a_2+20a_1^2a_4^2\
&quad-20a_1a_2a_3a_4-20a_1a_3^3-20a_2^3a_4+20a_2^2a_3^2+40a_1^2a_3+40a_1a_2^2+10a_2a_4^2\
&quad +10a_3^2a_4-30a_1a_4-30a_2a_3+13.
endalign



However, it seems no hope for determination of $a_4$.






share|cite|improve this answer














As some people said, the 2nd formula is easy to derive. In Maple, there are commands to get the minimal polynomial of LHS of the 2nd. The following method works for the 2nd formula, but not for the 3rd.



Prove that $sqrt[3]cosfrac2pi7 + sqrt[3]cosfrac4pi7 + sqrt[3]cosfrac6pi7 = sqrt[3]frac5-3sqrt[3]72$.



$textitProof$: $cosfrac2pi7,cosfrac4pi7,cosfrac6pi7$ are the three distinct real roots of $8x^3+4x^2-4x-1=0$.
Let $u_1=sqrt[3]cosfrac2pi7, u_2=sqrt[3]cosfrac4pi7, u_3=sqrt[3]cosfrac6pi7$ which are all real.
All roots of $8u^9+4u^6-4u^3-1=0$ are $u_1, t u_1, t^2 u_1, u_2, t u_2, t^2 u_2, u_3, t u_3, t^2 u_3$
where $t = e^2pi i/3$.



Let $(u-u_1)(u-u_2)(u-u_3)=u^3+a_2u^2+a_1u - 1/2$. We have
beginalign
(u-tu_1)(u-tu_2)(u-tu_3) &=u^3+ta_2u^2+t^2a_1u - 1/2, \
(u-t^2u_1)(u-t^2u_2)(u-t^2u_3) &=u^3+t^2a_2u^2+ta_1u - 1/2.
endalign
It follows from
beginalign
&8u^9+4u^6-4u^3-1 \
= & 8(u^3+a_2u^2+a_1u - 1/2)(u^3+ta_2u^2+t^2a_1u - 1/2)(u^3+t^2a_2u^2+ta_1u - 1/2)
endalign that
beginalign
a_2^3-3a_1a_2-2 &=0, \
4a_1^3+6a_1a_2+5 &= 0.
endalign
From these two equations, we get $a_1 = fraca_2^3-23a_2$
and $4a_2^9+30a_2^6+75a_2^3-32=0$.
Let $f(v)=4v^9+30v^6+75v^3-32$. We have $f'(v)=9v^2(2v^3+5)^2ge 0$. Thus, $f(v)=0$ has a unique real root.
It is easy to check that $v=-sqrt[3]frac5-3sqrt[3]72$ is a root of
$f(v)=0$. Thus, $a_2 = -sqrt[3]frac5-3sqrt[3]72$.
Thus, $u_1+u_2+u_3 = -a_2 = sqrt[3]frac5-3sqrt[3]72$. This completes the proof.



In general, suppose $x_1,x_2,x_3$ are the three distinct real roots of $x^3+bx^2+cx+d^3=0$,
where $b,c,d$ are rational numbers. We have
beginequation
sqrt[3]x_1+sqrt[3]x_2+sqrt[3]x_3 = sqrt[3]
-frac32Big(sqrt[3]Q+4sqrtDelta+sqrt[3]Q-4sqrtDeltaBig) -b-6d ,
endequation
where $Q = 24bd^2+36d^3+4bc+24cd, Delta = -4b^3d^3-27d^6+18bcd^3+b^2c^2-4c^3$.
Here, $Delta$ is the discriminant of $x^3+bx^2+cx+d^3=0$.



For the 3rd, similarly,
$x_k = cosfrac2kpi11, k=1,cdots, 5$ are the five distinct real roots of $32x^5+16x^4-32x^3-12x^2+6x+1=0$.
Let $u_1 = sqrt[5]cosfrac2pi11, u_2= sqrt[5]cosfrac4pi11, u_3 = sqrt[5]cosfrac6pi11, u_4 = sqrt[5]cosfrac8pi11, u_5= sqrt[5]cosfrac10pi11$.
All roots of $32u^25+16u^20-32u^15-12u^10+6u^5+1=0$ are
$t^j u_k, k=1,2,3,4, 5; j=0,1,2,3,4$ where $t= e^2pi i/5$.



Let $prod_k=1^5 (u-u_k) = u^5 + a_4u^4+a_3u^3+a_2u^2+a_1u+frac12$.
Similarly, we have four equations of
beginalign
0 & = a_4^5-5a_3a_4^3+5a_2a_4^2+5a_3^2a_4-5a_1a_4-5a_2a_3+2, \
0 & = 8a_1^5-20a_1^3a_2+10a_1^2a_3+10a_1a_2^2-5a_1a_4-5a_2a_3+1,\
0 & = -10a_1a_2a_4^3+10a_1a_3^2a_4^2+10a_2^2a_3a_4^2-10a_2a_3^3a_4+2a_3^5+10a_1^2a_4^2-10a_1a_2a_3a_4\
&quad-10a_1a_3^3-10a_2^3a_4+10a_2^2a_3^2-5a_3a_4^3+10a_1^2a_3+10a_1a_2^2+10a_2a_4^2+10a_3^2a_4\
&quad -15a_1a_4-15a_2a_3+7,\
0 & = -40a_1^3a_3a_4+40a_1^2a_2^2a_4+40a_1^2a_2a_3^2-40a_1a_2^3a_3+8a_2^5-40a_1^3a_2+20a_1^2a_4^2\
&quad-20a_1a_2a_3a_4-20a_1a_3^3-20a_2^3a_4+20a_2^2a_3^2+40a_1^2a_3+40a_1a_2^2+10a_2a_4^2\
&quad +10a_3^2a_4-30a_1a_4-30a_2a_3+13.
endalign



However, it seems no hope for determination of $a_4$.







share|cite|improve this answer














share|cite|improve this answer



share|cite|improve this answer








edited Aug 17 at 3:53

























answered Aug 17 at 2:53









River Li

193




193







  • 2




    I don't mean to be rude, by I find this utter unreadable. Could you perhaps make use of some displayed math environments (using $$ (math here) $$) or the align environment (beginalign endalign) to make it easier to follow? You can also find other formatting tips in the MathJax tutorial.
    – Xander Henderson
    Aug 17 at 3:19












  • 2




    I don't mean to be rude, by I find this utter unreadable. Could you perhaps make use of some displayed math environments (using $$ (math here) $$) or the align environment (beginalign endalign) to make it easier to follow? You can also find other formatting tips in the MathJax tutorial.
    – Xander Henderson
    Aug 17 at 3:19







2




2




I don't mean to be rude, by I find this utter unreadable. Could you perhaps make use of some displayed math environments (using $$ (math here) $$) or the align environment (beginalign endalign) to make it easier to follow? You can also find other formatting tips in the MathJax tutorial.
– Xander Henderson
Aug 17 at 3:19




I don't mean to be rude, by I find this utter unreadable. Could you perhaps make use of some displayed math environments (using $$ (math here) $$) or the align environment (beginalign endalign) to make it easier to follow? You can also find other formatting tips in the MathJax tutorial.
– Xander Henderson
Aug 17 at 3:19












 

draft saved


draft discarded


























 


draft saved


draft discarded














StackExchange.ready(
function ()
StackExchange.openid.initPostLogin('.new-post-login', 'https%3a%2f%2fmath.stackexchange.com%2fquestions%2f612607%2fwhat-comes-after-cos-left-tfrac2-pi7-right1-3-cos-left-tfrac4-pi%23new-answer', 'question_page');

);

Post as a guest













































































這個網誌中的熱門文章

Is there any way to eliminate the singular point to solve this integral by hand or by approximations?

Why am i infinitely getting the same tweet with the Twitter Search API?

Carbon dioxide